Infinite Series (Integral Test)

Click For Summary
The discussion centers on evaluating the convergence of the series ∑ n=1 ∞ (e^n/(e^(2n)+9)) using the integral test. The integral evaluation leads to the expression (1/3)tan-1(e∞/3), which simplifies to (1/3)(nπ/2) for odd n, raising questions about the convergence proof. Participants clarify that the lower limit of the integral must be considered, indicating that the definite integral does not yield multiple values. Additionally, it is pointed out that the function being integrated should account for the series starting at n=1, suggesting that the integral of f(x)=(e^x+9e^{-x})^{-1} over (1,∞) should produce two terms. The discussion emphasizes the importance of correctly applying the integral test to determine convergence.
Fernando Rios
Messages
96
Reaction score
10
Homework Statement
Use the integral test to find whether the following series converge or diverge.
Relevant Equations
∑ n=1 ∞ (e^n/(e^(2n)+9))
After evaluating the integral I found the following:

(1/3)tan-1(e/3) = (1/3)tan-1(∞) = (1/3)(nπ/2), where n is an odd number. In this case I found multiple solutions to the problem. How do you prove it converges?
 
Physics news on Phys.org
Hopefully, you aren't confusing the n in your answer with the n in the series. Just use ##n=1## in your answer and you are done.
 
Remember there's a lower limit on the integral as well, so you don't actually end up with multiple values for the definite integral.
 
Fernando Rios said:
Homework Statement: Use the integral test to find whether the following series converge or diverge.
Homework Equations: ∑ n=1 ∞ (e^n/(e^(2n)+9))

After evaluating the integral I found the following:

(1/3)tan-1(e∞/3) = (1/3)tan-1(∞) = (1/3)(nπ/2)

This is very much irrelevant to your conclusion, but isn't there supposed to be another term in your answer? Your infinite series starts at ##n=1##. So if each term in your summation is of the form ##f(n)=(e^n+9e^{-n})^{-1}##, you'd have to be integrating the function ##f(x)=(e^x+9e^{-x})^{-1}## over the interval ##(1,\infty)##, which should produce two terms. And as I recall, the standard ##\arctan## function is bounded between ##(-\pi/2,\pi/2)##, so I cannot see how ##n## cannot be anything other than ##n=1##.
 
Last edited:
Question: A clock's minute hand has length 4 and its hour hand has length 3. What is the distance between the tips at the moment when it is increasing most rapidly?(Putnam Exam Question) Answer: Making assumption that both the hands moves at constant angular velocities, the answer is ## \sqrt{7} .## But don't you think this assumption is somewhat doubtful and wrong?

Similar threads

  • · Replies 1 ·
Replies
1
Views
2K
  • · Replies 7 ·
Replies
7
Views
2K
  • · Replies 16 ·
Replies
16
Views
4K
  • · Replies 3 ·
Replies
3
Views
1K
  • · Replies 6 ·
Replies
6
Views
2K
  • · Replies 2 ·
Replies
2
Views
1K
  • · Replies 4 ·
Replies
4
Views
2K
Replies
14
Views
2K
  • · Replies 2 ·
Replies
2
Views
2K
  • · Replies 7 ·
Replies
7
Views
2K